Merge remote branch 'public/master'
[course.git] / latex / problems / Serway_and_Jewett_4 / problem19.16.tex
1 \begin{problem*}{19.16}
2 Consider the electric dipole shown in Figure P19.16.  Show that the
3 electric field at a distant point on the $+x$ axis is $E_x \approx
4 4k_eqa/x^3$.
5 \end{problem*} % problem 19.16
6
7 \empaddtoprelude{
8   pair A, B;
9   numeric a;
10   a := 1cm;
11   A := (-a,0);
12   B := (a, 0);
13   C := (6a, 0);
14   def drawC = 
15     drawarrow (A-(a,0))--(C+(a,0)) withpen pencircle scaled 0pt;
16     draw_ncharge(A, 6pt);
17     draw_pcharge(B, 6pt);
18     label.top("0", draw_ltic(origin, 90, 0, 3pt, 0pt, black));
19     dotlabel.bot("x", C);
20     label.bot("a", draw_length(A, origin, 10pt));
21     label.bot("a", draw_length(origin, B, 10pt));
22     labeloffset := 8pt;
23     label.top("-q", A);
24     label.top("q", B);
25   enddef;
26 }
27
28 \begin{nosolution}
29 \begin{center}
30 \begin{empfile}[3p]
31 \begin{emp}(0cm, 0cm)
32   drawC;
33 \end{emp}
34 \end{empfile}
35 \end{center}
36 \end{nosolution}
37
38 \begin{solution}
39 \begin{center}
40 \begin{empfile}[3]
41 \begin{emp}(0cm, 0cm)
42   label.top(btex $E_{q}$ etex, draw_Efield(B, C, 18pt));
43   label.top(btex $E_{-q}$ etex, draw_Efield(A, C, -16pt));
44   drawC;
45 \end{emp}
46 \end{empfile}
47 \end{center}
48 Let us assume the point in question has a positive $x$ value (just
49 reverse the sign if $x < 0$).
50 \begin{equation}
51  \vect{E} = k_e \sum_i \frac{q_i}{r_i^2}\rhat_i
52           = k_e \left[\frac{q}{(x-a)^2}\ihat + \frac{-q}{(x+a)^2}\ihat\right]
53 \end{equation}
54 For $|x| \gg |c|$,
55 \begin{equation}
56  (x+c)^n = x^n \left(1+\frac{c}{x}\right)^n
57          = x^n \left[1 + n\frac{c}{x} + \frac{n(n-1)}{2}\cdot\left(\frac{c}{x}\right)^2 + \ldots\right]
58          \approx x^n (1 + n\frac{c}{x}) \;,
59 \end{equation}
60 because $(c/x)^2$ is very, very small.  (We are Taylor expanding
61 $(x+c)^n$ as a function of $c/x$, and keeping only the first two
62 terms.)  In our case, $n = -2$ and $c = \mp a$
63 \begin{equation}
64  \vect{E} = k_e \left[\frac{q}{x^2}\left(1-2\frac{-a}{x}\right) + \frac{-q}{x^2}\left(1-2\frac{a}{x}\right)\right]\ihat
65           = k_e \frac{q}{x^2}\left(1+2\frac{a}{x} - 1+2\frac{a}{x}\right)\ihat
66           = \ans{\frac{4 k_e q a}{x^3}\ihat}
67 \end{equation}
68 \end{solution}